I need y'all help so bad

I Need Y'all Help So Bad

Answers

Answer 1

Answer:

30, 15, 50, 20

Step-by-step explanation:

I gotchu

median is found by doing this;

(see first image)

So, for data set A, it would be:

*Btw im assuming the numbers are the same on both data sets since there is the same amount of lines in each and the same space btwn the lines*

10,10,20,20,20,30,30,30,30,40,40,50

Median is: 30

Iqr is found by doing this:

(see second image)

So for data set A, it would be:

IQR:15

---------------------------------------------------------

Set B(same thing)

Median:

30,40,40,40,50,50,50,50,60,60,70,70

Median is: 50

IQR: 20

That should be all! Let me know if I'm correct ;)

I Need Y'all Help So Bad
I Need Y'all Help So Bad

Related Questions

Do what it says on the image

Answers

Answer:

Because every Y doesn't have 2 different X

Step-by-step explanation:

Which point is not on the graph of the function y = x + 10?

(8, 18)
(3, 13)
(20, 10)
(482, 492)

Answers

is there any pic of the graph?

Answer:

(20, 10)

Step-by-step explanation:

y = x + 10

(8, 18)

y = x + 10

18 = 8 + 10

18 = 18 (this point is one the graph)

(3, 13)

y = x + 10

13 = 3 + 10

13 = 13 (this point is on the graph)

(20, 10)

y = x + 10

10 = 20 + 10

10 = 30 (this point is not on the graph)

(482, 492)

y = x + 10

492 = 482 + 10

492 = 492 (this point is on the graph)

Hope this helps!

The dot plots below show the scores for a group of students who took two rounds of a quiz:

Two dot plots are shown one below the other. The title for the dot plot on the top is Round 1 and the title for the bottom plot is Round 2. Below the line for each dot plot is written Score. There are markings from 5 to 9 on the line at intervals of one. There are 2 dots above the mark 5, 3 dots above the mark 6, 4 dots above the mark 7, and 1 dot above the mark 8. For the bottom dot plot there are 6 dots above the mark 6, 2 dots above the mark 8, and 2 dots above the mark 9.

Which of the following inferences can be made using the dot plot?

The range of each round is the same.
There is no overlap between the data.
Round 1 scores were higher than round 2 scores.
Round 2 scores were lower than round 1 scores.

Answers

Answer:

The inference that can be made using the dot plot is:

   The range of round 1 is greater than the round 2 range

Step-by-step explanation:

Round 1:

Score                Frequency

 1                          0

 2                          2

 3                          3

 4                          2  

 5                          1

Hence, the minimum score of Round 1 is: 2

maximum score is: 5

Hence, Range=Maximum value-Minimum score

                     =5-2

                      =3

Similarly, Round-2

Score                Frequency

 1                          0

 2                         0

 3                          0

 4                          4  

 5                          4

Hence, the minimum score of Round 1 is: 4

maximum score is: 5

Hence, Range=Maximum value-Minimum score

                     =5-4

                      =1

The scores of round 2 are higher than round-1.

Since round 2 have a higher frequency for higher scores as compared to round-1.

Hence, Range of round 1 is greater than the range of Round-2.

Can someone help me on 7,8, and 9 so confused

Answers

Step-by-step explanation:

all you have to do is multiply all those numbers together so 0.5 * 4.2 * 2.8 *7 =41.16

the process the exact same for all the other ones they have all your stuff set out for you on each one of the shapes

What is the median of the data set represented by the dot plot?


Enter the answer in the box.

Answers

Answer:

The median represent the middle value of a data set when it is placed in order from smallest to largest.

Answer:14
Explanation: there are equal numbers of dots on the left hand right sides of the number 14.
Therefore, 14 is in the middle.

Hopes It Helps
Brainliest pls :)

BRAINLIST+DONT SCAM ME
The number of marbles of different colors stored in a hat is listed below:


1 red marble

8 green marbles

9 blue marbles

Without looking in the hat, Ray takes out a marble at random. He replaces the marble and then takes out another marble from the hat. What is the probability that Ray takes out a blue marble in both draws?

fraction 1 over 9

fraction 1 over 8

fraction 1 over 4

fraction 1 over 2

Answers

Answer:

1/4

Step-by-step explanation:

1 red marble, 8 green marbles. 9 blue marbles

There are 1+8+9 = 18 marbles

First marble blue

P(blue) = blue / total = 9/18 = 1/2

He replaces the marble so there are still 18 marbles

1 red marble, 8 green marbles. 9 blue marbles

There are 1+8+9 = 18 marbles

Second marble blue

P(blue) = blue / total = 9/18 = 1/2

Multiply the probabilities together

P(blue, replace, blue) = 1/2 * 1/2 = 1/4

Sorry Im a little late The correct answer is Infact C. Fraction 1 over 4 (Looks like this [tex]\frac{1}{4\\}[/tex])I Indeed took the test On FLVS. 8.13  segment exam pt.2. Proof.

Step-by-step explanation:

Have a great Day!

Solve the systems of equations by elimination.
7×-3y=-37
5×=3y-11

6y=26-5x
X+6y=10

Answers

Answer:

1. (-13, -18) 2. (4, 1)

Step-by-step explanation:

Add the equations in order to solve for the first variable. Plug this value into the other equations in order to solve for the remaining variables.

Answer:

(-13, -18)

Step-by-step explanation:

x = -13

y = -18

!!!!!!!!!!!!!!!!!!!!!!!!!!!!!!!!!!!!!!!! !!!!!!!!!!!!!!!!!!!!!!!!

Answers

Answer:

[tex]62\\ R30[/tex]

Solve:

86 goes into 5,362, 6 times

The result is 516.

Drop the 6 from the tens column of the dividend.

516 - 536 = 20

Drop 6 again, this time from the ones place.

86 goes into 202, 2 times

The result is 172.

202 - 172 = 30 remainder

Answer:

The answer is in the photo below :)

Your welcome.

Sorry if it's wrong.

Bye. have a great day/night

You invest $950 and over 4 years you earn $266. What is the interest rate?

Answers

Answer:

7% per year

Step-by-step explanation:

r = rate
t = time (in years)
a = principal + interest (total amount)
p = principal amount
formula r = (1/t)(A/P - 1)

r = (1/4)((1216/950) - 1) = 0.07

r = 0.07

Converting r decimal to R a percentage

R = 0.07 * 100 = 7%/year

Answer:

0.07 = 7%

Step-by-step explanation:

Which of the following ratios represents this sentence: In the ocean, for every 1 kilogram of water, there are 35,000 milligrams of salt.
A Ocean:1:35,000
B 35,000 - 1
C 35,000:Ocean to 1
D 1 to 35,000

Answers

Answer:

C because if 1 kilogram is 35000 that mean 3500 To 1

Answer :

C) 35,000:ocean to 1

Answer me properly then get brainliest!!!!!!!!!!
edit this picture and write the correct answers

Answers

Say: The smaller box plot is more accurate because the data is closer together. Also work out the difference between the medians. Hope this helps :)
Small box is more acurrate

I needs help please, a step by step answer

Answers

Answer:

68

Step-by-step explanation:

Pythagorean theorem: a²+b²=c²  a²=32²   b²=60²   c²=hypotenuse²

32²+60²=c²

1024+3600= c²

4624=c²

Now, take the square root of both sides of the equation to find c

[tex]\sqrt{4624}[/tex]=c

68=c

The answer to this question is 68.

Which expression represents the statement "12 divided by 3, then add 5"?

A. 12÷ (3+5)
B. 3 ÷ 12 + 5
C. 12÷3+5
D. 3+12 ÷ 5

Answers

Answer:

C!

Step-by-step explanation:

Answer:

I would say A or C, most likely A

Step-by-step explanation:

Please help me!!!!!! asap- thx <333

Answers

Answer:

x+4=6.5

x=2.5

Step-by-step explanation:

It says x increased by 4 which means x+4. It also says x is equal to 6.5 so we write x+4=6.5

Now we solve for x.

x+4=6.5

we can get x by itself by removing the 4

x+4-4=6.5-4

we have to do the same thing to each side to keep the equation equal.

now we can simplify.

x=2.5

Answer:

X+4=6.5

x=2.5

Step-by-step explanation:

To find value of x, just subtract:

6.5-4=2.5

So the value of x is 2.5

This pattern follows the "multiply by 3" rule. What is the next number in the pattern?

3, 9, 27, 81, ________

Group of answer choices

243

244

288

343

Answers

243 you multiply 81 times 3

Answer:

243 because 81*3=243

Step-by-step explanation:

81 times 3 = 243

What is the answer to this? Please answer correctly if your answer is incorrect then I will not accept it.

Answers

There are not similar because they are different triangles one is a right triangle in one as a acute triangle

Dan has ridden 33 miles of a bike course. The course is 60 miles long. What percentage of the course has Dan ridden so far?

Answers

Answer:

55%

Step-by-step explanation:

When we are using percentages, what we are really saying is that the percentage is a fraction of 100. "Percent" means per hundred, and so 50% is the same as saying 50/100 or 5/10 in fraction form.

So, since our denominator in 33/60 is 60, we could adjust the fraction to make the denominator 100. To do that, we divide 100 by the denominator:

100 ÷ 60 = 1.6666666666667

Once we have that, we can multiple both the numerator and denominator by this multiple:

33 x 1.6666666666667

60 x 1.6666666666667

=

55

100

Now we can see that our fraction is 55/100, which means that 33/60 as a percentage is 55%.

We can also work this out in a simpler way by first converting the fraction 33/60 to a decimal. To do that, we simply divide the numerator by the denominator:

33/60 = 0.55

Once we have the answer to that division, we can multiply the answer by 100 to make it a percentage:

0.55 x 100 = 55%

He has ridden 55% of the course. You have to divide 33 by 60 to get .55
Then you just turn the decimal .55 into a percentage which is 55%

The circumference of the smaller circle is 30% of the circumference of the larger circle. Find the circumference of the larger circle. Round your answer to the nearest tenth.

Answers

Answer:

The circumference of the larger circle is 94.2 inches.

Step-by-step explanation:

Answer:

simply 94.2

Step-by-step explanation:

Answer this please and thank you

Answers

Answer:

2/9

Step-by-step explanation:

1) Use the first fraction rule, which is -a/-b=a/b, witch gives you

-10/21 / 15/7

2. Use the second fraction rule, which is a*d/b*c

Using a*d, b*c, you would get 10 x 7/21 x 15= 70/315

3. Simply our answer

Divide 70/135 by 35, witch would be 70 divided by 35/ 315 divided by 35, witch would be 2/9 , because 70 divided by 35 is 2 and 315 divided by 35 is 9.

Step-by-step explanation:

-10 ÷ -15

21. 7

-10 × 7

21. 15

-2 ×. 1

3. 3

-2

9

please help the last person's answer was wrong. will mark brainliest.

Answers

363 in^2

The radius is 11 (as shown in the diagram).

A = pi(r)^2

pi = 3

3(11)^2 = 3*121 = 363 in^2

Which line plot displays a data set with an outlier?

Answers

Answer:

The first plot.

Step-by-step explanation:

You're looking for a plot that has a dot that's very far apart from the rest of the dots. The first plot has exactly that.

The first plot

In these type of questions you’re looking for a point that stands out from the rest as in the numbers (2,4,6,8,100) 100 would be the outlier.

Hurry
Which inequality does this graph show?

Answers

Answer:

y>0 mean the upper side of your or the positive ones [1,2,3,...] are the ans

y>or equal 0 means the whole numbers

The answer is A. it is y>0

use the distributive property to remove the parenthesis

(problem is below)

Answers

Answer:

6w-66

Step-by-step explanation:

We can use the distributive property by multiplying the number outside the parentheses to each term inside the parentheses. So we have (w-11)6 = 6*w-6*11 = 6w-66.  

Find the value of [tex]\frac{2}{3} ^{3}[/tex]. Explain or show how you got your answer.

Answers

Answer:

2/3³= 2/( 3 x 3 x 3 ) = 2 / 27

Step-by-step explanation:

What's 34x9(4+27x3)x6+32

Answers

Answer:

156092

Step-by-step explanation:

34x9(4+27x3)x6+32

(4+27x3) = (4+81) = (85)

34x9 = 306

Now we have: 306(85)x6+32

306(85) = 26010 =>> 26010x6+32

26010x6= 156060+32= 156092

Answer:

156092

Step-by-step explanation:

34x9(4+27x3)x6+32 = 156092

PLEASE HELP ME AFAP!

The scatter plot below shows the number of pizzas sold during weeks when different numbers of coupons were issued. The equation represents the linear model for this data.



y = 3.4x + 43



According to the model, how many pizzas will be sold nightly if 15 coupons are issued?

Answers

Answer:

95 pizzas i think i honestly don't know I'm really sorry if I'm wrong

Step-by-step explanation:

sorryyy

The answer to this is 95 pizzas

From the set {8.13, 12.32, 17.44}, use substitution to determine which value(s) of x makes the inequality true. x > 12.32

Answers

Answer:

17.44 > 12.32

Step-by-step explanation:

From the set {8.13, 12.32, 17.44}, use substitution to determine which value(s) of x makes the inequality true. x > 12.32

8.13:

x > 12.32

8.13 > 12.32

False

12.32:

x > 12.32

12.32 > 12.32

False

17.44:

x > 12.32

17.44 > 12.32

True

There, our final answer is 17.44

Hope this helps!

17.44 is the answer I think :)

Bethany is building storage boxes. Each box will have a length of 6 feet, a width of 312 feet, and a height of 512 feet. A quart of paint covers 100 square feet.

How many quarts of paint does Bethany need to buy in order to paint 5 boxes?


4 quarts

6 quarts

7 quarts

8 quarts

Answers

Answer:

8 quarts

Step-by-step explanation:

first, we need to find the surface area by doing 2×(6×3.5 + 6×5.5 + 3.5×5.5) = 146.5 feet(2  now we see that we have 5 total boxes so we multiply 146.5x5=732.5 1q=100 square feet so 700=7q but we still have that 32.5 left for we need 8q because it's better to have more than coming up short

Hope this helps:)

The number of quarts of paint Bethany needs to buy in order to paint 5 boxes is 7. The correct option is C.

What is Geometry?

It deals with the size of geometry, region, and density of the different forms both 2D and 3D.

Bethany is building storage boxes. Each box will have a length of 6 feet, a width of 3.12 feet, and a height of 5.12 feet. A quart of paint covers 100 square feet.

The surface area of the box will be

[tex]\rm Surface \ area = 2(lb + bh + hl)\\\\Surface \ area = 2(6*3.12 + 3.12*5.12 + 5.12 *6)\\\\Surface \ area = 2(18.72 +15.9744 + 30.72)\\\\Surface \ area = 2*65.4144\\\\Surface \ area = 130.8288[/tex]

There are 5 boxes, then the surface area of 5 boxes will be

Surface area = 130.8288 × 5 = 654.144

A quart of paint covers 100 square feet.

Then the number of quarts required will be

[tex]\rm Number \ o f \ quart = \dfrac{654.144}{100}\\\\Number \ o f \ quart = 6.54144 \approx 7[/tex]

More about the geometry link is given below.

https://brainly.com/question/7558603

Pls, help!!!! Will make brainliest!!! ANSWER BOTH PARTS!!!

Answers

Answer:

y= -45x + 380 based on how many pages are left or

y= 45x based on how much he read

Step-by-step explanation:

Anyways, lets solve with the first formula

1. A week is seven days, so substitute 7 as x.

y= -45(7)+380

-45 (7) = -45 x 7 = -315

-315+380= 65

He has 65 pages left after the week which means he has not finished the book

2. The Formula

Use the formula y=mx+b

m is the slope or the rate of change. What is affecting the constant number of 380?

The 45 pages he is reading change how many pages he has left right?

That means the 45 that is changing b is the slope or m, but it is negative because its affecting how many pages he has left in a negative way.

b is the starting value

That is 380 pages left, which is what he starts off as the slope

Hence the equation is

y=-45x+380

Answer: y= -45x + 380 based on how many pages are left or

y= 45x based on how much he read

Step-by-step explanation:  1. A week is seven days, so substitute 7 as x.

y= -45(7)+380

-45 (7) = -45 x 7 = -315  2. The Formula

Use the formula y=mx+b

m is the slope or the rate of change. What is affecting the constant number of 380?

The 45 pages he is reading change how many pages he has left right?

That means the 45 that is changing b is the slope or m, but it is negative because it's affecting how many pages he has left negatively.

b is the starting value

That is 380 pages left, which is what he starts as the slope

Hence the equation is

y=-45x+380

An ordinary (fair) die is a cube with the numbers 1 through 6 on the sides (represented by painted spots). Imagine that such a die is rolled twice in succession and that the face values of the two rolls are added together. This sum is recorded as the outcome of a single trial of a random experiment. Compute the probability of each of the following events. Event A: The sum is greater than 6. Event B: The sum is not divisible by 3. Write your answers as fractions.

Answers

Answer:

11

Step-by-step explanation:

if 6+5  and its not divisblibe by 3

The probability of the sum being greater than 6 will be 7/12. And the probability of the sum is not divisible by 3 will be 1/3.

What is probability?

Its basic premise is that something will almost certainly happen. The percentage of favorable events to the total number of occurrences.

An ordinary (fair) die is a cube with the numbers 1 through 6 on the sides (represented by painted spots).

Imagine that such a die is rolled twice in succession and that the face values of the two rolls are added together.

This sum is recorded as the outcome of a single trial of a random experiment.

The number of the total events will be

Total events = 6 x 6

Total events = 36

The total sample is given below.

(1,1), (1,2), (1,3), (1,4), (1,5), (1,6)

(2,1),(2,2),(2,3),(2,4),(2,5),(2,6)

(3,1),(3,2),(3,3),(3,4),(3,5),(3,6)

(4,1),(4,2),(4,3),(4,4),(4,5),(4,6)

(5,1),(5,2),(5,3),(5,4),(5,5),(5,6)

(6,1),(6,2),(6,3),(6,4),(6,5),(6,6)

The probability of the sum is greater than 6 will be

Favorable events = 21

P = 21 / 36

P = 7/12

The probability of the sum is not divisible by 3 will be

Favorable events = 21

P = 12 / 36

P = 1/3

More about the probability link is given below.

https://brainly.com/question/795909

#SPJ5

Other Questions
The magnitude, m, of an earthquake is defined to be m = log startfraction i over s endfraction, where i is the intensity of the earthquake (measured by the amplitude of the seismograph wave) and s is the intensity of a "standard" earthquake, which is barely detectable. what is the magnitude of an earthquake that is 35 times more intense than a standard earthquake? use a calculator. round your answer to the nearest tenth. 1.5 0.5 1.5 3.6 Are these shapes similar? Choose all that apply.No, the corresponding angles are not congruent.Yes, the corresponding sides are proportional.Yes, the corresponding angles are all congruent.No, the corresponding sides are not proportional. Mrs. Fatima has a a small sports car that she uses to drop her kids off from school that has a mass of 5000 kg. Mr. Zaid drives a truck with a mass of 4200 kg. If the two vehicles are travelling down the road, side by side, at a speed of 75 km/hr, which one will have the most momentum? Explain* 1.write down a list that define daily life in the digital age from computers and websites to smart phones in your list explain. How each item affect daily in modern age 2 think about which item from list best define daily life in the digital age and why3.based on your thoughts look at your list and choose the three items you think are important 2. Nous _____________________________ (faire) un pique-nique dans la fort. (Futur proche) can someone help me with this question please. June is driving from brookline, massachusetts to brooklyn, new york. The cities are 200 miles apart. Pretend june lives in a world in which she encounters no traffic jams and can drive at a constant speed of 50mph the entire way. Consider the following function. the distance june has traveled, c, is a function of her driving time, t.write a formula that describes this function PLEASE HELP I REALLY REALLY NEED HELP I'll literally give someone brainlest if they get the question right How do I create a teacher account? Which answers describe the shape below? Check all that apply. A. Parallelogram B. Quadrilateral O C. Trapezoid D. Rhombus E. Rectangle F. Square What improves the productivity of cellular respiration?. HELPPPP Select the correct answer.What explains the Trump administrations reason for reversing President Obama's Clean Power Plan?A. They wanted to lower the price of oil exported to other countries.B. They believed the United States needed to become more energy independent. C. They wanted to increase costs for the oil companies.D. They believed the presidents plan would reduce a higher percentage of emissions. Compare and contrast the consequences of German aggression and Japanese aggression by other world powers. When German attacked Rhineland and When japan attacked Manchuria Pls help me ^^Scientists use radiometric dating to determine the age ofrocks. Radioactive atoms are used for radiometric dating.Which term also describes dating using radioactive atoms?A. Atomic datingB. Chemical datingC. Absolute datingD. Relative dating A laundry basket contains 8 socks, of which 2 are blue. Analyze Star Streams cost-volume-profit relationships Star Stream is a subscription-based video streaming service. Subscribers pay $120 per year for the service. Star Stream licenses and develops content for its subscribers. In addition, Star Stream leases servers to hold this content. These costs are not variable to the number of subscribers, but must be incurred regardless of the subscriber base. In addition, Star Stream compensates telecommunication companies for bandwidth so that Star Stream customers receive fast streaming services. These costs are variable to the number of subscribers. These and other costs are as follows: Enter your answers in whole dollars. Assume the same content cost scenario as in (b). How much would the annual subscription need to change in order to maintain the same break-even as in (a)? The annual subscription need to Accounts receivable is classified on the balance sheet as aa. current asset.b. current liability.c. noncurrent asset.d. long-term liability. The ______________ did not agree with this and instead decided they no longerwould be a part of the United States at all--they would ______________. In1861, just when President Lincoln was taking office, seven southern states left theUnion and started their own new country: the __________________________of America. Before long, four more states joined them in leaving the Union. Answer correct with no website the correct answer When an individual carries more accident and health insurance than he/she would need for a loss, it is called: The points K(5,-5), L(9,-5), and M (7,3)